Physics, asked by pinky221122, 11 months ago

answer the following question in picture

Attachments:

pinky221122: plz answer
pinky221122: the 3rd question

Answers

Answered by ananyaanuska
0
for first question use the conservation of momentum formula... i.e. m1v1=m2v2
Similar questions